Is it based on Tchebyshev?











up vote
2
down vote

favorite












If $a$, $b$, $c$ are positive real numbers such that $a^2 + b^2 + c^2 = 27$. Find the least value of $a^3 + b^3 + c^3$ ?



I tried with Tchebyshev inequality on sets ${a, b, c}$ and ${a^2, b^2 , c^2}$ but was blocked. I also tried with mean of $m-$the power with $m=3$ but again could not proceed further.



I only know elementary inequalities like Cauchy-Schwarz and AM-GM apart from above mentioned inequalities. Please help.










share|cite|improve this question




























    up vote
    2
    down vote

    favorite












    If $a$, $b$, $c$ are positive real numbers such that $a^2 + b^2 + c^2 = 27$. Find the least value of $a^3 + b^3 + c^3$ ?



    I tried with Tchebyshev inequality on sets ${a, b, c}$ and ${a^2, b^2 , c^2}$ but was blocked. I also tried with mean of $m-$the power with $m=3$ but again could not proceed further.



    I only know elementary inequalities like Cauchy-Schwarz and AM-GM apart from above mentioned inequalities. Please help.










    share|cite|improve this question


























      up vote
      2
      down vote

      favorite









      up vote
      2
      down vote

      favorite











      If $a$, $b$, $c$ are positive real numbers such that $a^2 + b^2 + c^2 = 27$. Find the least value of $a^3 + b^3 + c^3$ ?



      I tried with Tchebyshev inequality on sets ${a, b, c}$ and ${a^2, b^2 , c^2}$ but was blocked. I also tried with mean of $m-$the power with $m=3$ but again could not proceed further.



      I only know elementary inequalities like Cauchy-Schwarz and AM-GM apart from above mentioned inequalities. Please help.










      share|cite|improve this question















      If $a$, $b$, $c$ are positive real numbers such that $a^2 + b^2 + c^2 = 27$. Find the least value of $a^3 + b^3 + c^3$ ?



      I tried with Tchebyshev inequality on sets ${a, b, c}$ and ${a^2, b^2 , c^2}$ but was blocked. I also tried with mean of $m-$the power with $m=3$ but again could not proceed further.



      I only know elementary inequalities like Cauchy-Schwarz and AM-GM apart from above mentioned inequalities. Please help.







      inequality






      share|cite|improve this question















      share|cite|improve this question













      share|cite|improve this question




      share|cite|improve this question








      edited Nov 16 at 13:05









      amWhy

      191k27223438




      191k27223438










      asked Nov 16 at 11:47









      Ramanujam Ganit Prashikshan Ke

      1267




      1267






















          4 Answers
          4






          active

          oldest

          votes

















          up vote
          1
          down vote



          accepted










          You can do it using the mean of the $m$ power for $m=frac{3}{2}$ (or convexity of $x^frac{3}{2}$ for that matter) :



          We have that $frac{x^{frac{3}{2}}+y^{frac{3}{2}}+z^frac{3}{2}}{3} geq (frac{x+y+z}{3})^frac{3}{2}$ for any $x,y,z > 0$



          Substituting $x=a^2, y=b^2, z=c^2$ we obtain $frac{a^3+b^3+c^3}{3} geq (frac{a^2+b^2+c^2}{3})^frac{3}{2} = 9^frac{3}{2}=27$ and therefore the sought minimum is $3cdot 27= 81,$, attainable if(and only if) all three numbers are equal to $3$.






          share|cite|improve this answer






























            up vote
            2
            down vote













            From the power mean inequality we know that



            $$sqrt[q]{sum_{i=1}^nw_ix_i^q} geq sqrt[p]{sum_{i=1}^nw_ix_i^p} quad forall p <q$$



            where $ sum_{i=1}^n w_i=1 $



            In your case $p=2, q=3$ and $n=3$. Hence,



            $$left(dfrac{a^3+b^3+c^3}{3}right)^{1/3}ge left(dfrac{a^2+b^2+c^2}{3}right)^{1/2}=3$$






            share|cite|improve this answer




























              up vote
              2
              down vote













              Okay, the question requires knowledge of the AM-GM inequality, which you do have.
              You need to apply it twice; first on $a^2,b^2,c^2$ and obtain an inequality on the product $abc$; then on $a^3,b^3,c^3$ and substitute $abc$.
              The answer is $81$.
              Hope this helps.






              share|cite|improve this answer






























                up vote
                0
                down vote













                For $a=b=c=3$ we obtain a value $81$.



                We'll prove that it's a minimal value.



                Indeed, we need to prove that $$a^3+b^3+c^3geq81,$$ which is true because
                $$a^3+b^3+c^3-81=sum_{cyc}(a^3-27)=$$
                $$=sum_{cyc}left(a^3-27-frac{9}{2}(a^2-9)right)=frac{1}{2}sum_{cyc}(a-3)^2(2a+3)geq0.$$
                Also, Holder helps:
                $$a^3+b^3+c^3=sqrt{frac{(a^3+b^3+c^3)^2(1+1+1)}{3}}geqsqrt{frac{(a^2+b^2+c^2)^3}{3}}=81.$$






                share|cite|improve this answer























                • From which source can I understand holder's inequality with elementary illustrations and exercises. You seem to be good at this.
                  – Ramanujam Ganit Prashikshan Ke
                  Nov 16 at 16:55










                • @Ramanujam Ganit Prashikshan Ke You can read solutions with tag holder-inequality in this forum. I think it's the best illustration. See also here: math.stackexchange.com/tags/holder-inequality/info
                  – Michael Rozenberg
                  Nov 16 at 17:29













                Your Answer





                StackExchange.ifUsing("editor", function () {
                return StackExchange.using("mathjaxEditing", function () {
                StackExchange.MarkdownEditor.creationCallbacks.add(function (editor, postfix) {
                StackExchange.mathjaxEditing.prepareWmdForMathJax(editor, postfix, [["$", "$"], ["\\(","\\)"]]);
                });
                });
                }, "mathjax-editing");

                StackExchange.ready(function() {
                var channelOptions = {
                tags: "".split(" "),
                id: "69"
                };
                initTagRenderer("".split(" "), "".split(" "), channelOptions);

                StackExchange.using("externalEditor", function() {
                // Have to fire editor after snippets, if snippets enabled
                if (StackExchange.settings.snippets.snippetsEnabled) {
                StackExchange.using("snippets", function() {
                createEditor();
                });
                }
                else {
                createEditor();
                }
                });

                function createEditor() {
                StackExchange.prepareEditor({
                heartbeatType: 'answer',
                convertImagesToLinks: true,
                noModals: true,
                showLowRepImageUploadWarning: true,
                reputationToPostImages: 10,
                bindNavPrevention: true,
                postfix: "",
                imageUploader: {
                brandingHtml: "Powered by u003ca class="icon-imgur-white" href="https://imgur.com/"u003eu003c/au003e",
                contentPolicyHtml: "User contributions licensed under u003ca href="https://creativecommons.org/licenses/by-sa/3.0/"u003ecc by-sa 3.0 with attribution requiredu003c/au003e u003ca href="https://stackoverflow.com/legal/content-policy"u003e(content policy)u003c/au003e",
                allowUrls: true
                },
                noCode: true, onDemand: true,
                discardSelector: ".discard-answer"
                ,immediatelyShowMarkdownHelp:true
                });


                }
                });














                 

                draft saved


                draft discarded


















                StackExchange.ready(
                function () {
                StackExchange.openid.initPostLogin('.new-post-login', 'https%3a%2f%2fmath.stackexchange.com%2fquestions%2f3001046%2fis-it-based-on-tchebyshev%23new-answer', 'question_page');
                }
                );

                Post as a guest















                Required, but never shown

























                4 Answers
                4






                active

                oldest

                votes








                4 Answers
                4






                active

                oldest

                votes









                active

                oldest

                votes






                active

                oldest

                votes








                up vote
                1
                down vote



                accepted










                You can do it using the mean of the $m$ power for $m=frac{3}{2}$ (or convexity of $x^frac{3}{2}$ for that matter) :



                We have that $frac{x^{frac{3}{2}}+y^{frac{3}{2}}+z^frac{3}{2}}{3} geq (frac{x+y+z}{3})^frac{3}{2}$ for any $x,y,z > 0$



                Substituting $x=a^2, y=b^2, z=c^2$ we obtain $frac{a^3+b^3+c^3}{3} geq (frac{a^2+b^2+c^2}{3})^frac{3}{2} = 9^frac{3}{2}=27$ and therefore the sought minimum is $3cdot 27= 81,$, attainable if(and only if) all three numbers are equal to $3$.






                share|cite|improve this answer



























                  up vote
                  1
                  down vote



                  accepted










                  You can do it using the mean of the $m$ power for $m=frac{3}{2}$ (or convexity of $x^frac{3}{2}$ for that matter) :



                  We have that $frac{x^{frac{3}{2}}+y^{frac{3}{2}}+z^frac{3}{2}}{3} geq (frac{x+y+z}{3})^frac{3}{2}$ for any $x,y,z > 0$



                  Substituting $x=a^2, y=b^2, z=c^2$ we obtain $frac{a^3+b^3+c^3}{3} geq (frac{a^2+b^2+c^2}{3})^frac{3}{2} = 9^frac{3}{2}=27$ and therefore the sought minimum is $3cdot 27= 81,$, attainable if(and only if) all three numbers are equal to $3$.






                  share|cite|improve this answer

























                    up vote
                    1
                    down vote



                    accepted







                    up vote
                    1
                    down vote



                    accepted






                    You can do it using the mean of the $m$ power for $m=frac{3}{2}$ (or convexity of $x^frac{3}{2}$ for that matter) :



                    We have that $frac{x^{frac{3}{2}}+y^{frac{3}{2}}+z^frac{3}{2}}{3} geq (frac{x+y+z}{3})^frac{3}{2}$ for any $x,y,z > 0$



                    Substituting $x=a^2, y=b^2, z=c^2$ we obtain $frac{a^3+b^3+c^3}{3} geq (frac{a^2+b^2+c^2}{3})^frac{3}{2} = 9^frac{3}{2}=27$ and therefore the sought minimum is $3cdot 27= 81,$, attainable if(and only if) all three numbers are equal to $3$.






                    share|cite|improve this answer














                    You can do it using the mean of the $m$ power for $m=frac{3}{2}$ (or convexity of $x^frac{3}{2}$ for that matter) :



                    We have that $frac{x^{frac{3}{2}}+y^{frac{3}{2}}+z^frac{3}{2}}{3} geq (frac{x+y+z}{3})^frac{3}{2}$ for any $x,y,z > 0$



                    Substituting $x=a^2, y=b^2, z=c^2$ we obtain $frac{a^3+b^3+c^3}{3} geq (frac{a^2+b^2+c^2}{3})^frac{3}{2} = 9^frac{3}{2}=27$ and therefore the sought minimum is $3cdot 27= 81,$, attainable if(and only if) all three numbers are equal to $3$.







                    share|cite|improve this answer














                    share|cite|improve this answer



                    share|cite|improve this answer








                    edited Nov 17 at 13:16









                    amWhy

                    191k27223438




                    191k27223438










                    answered Nov 16 at 13:32









                    Sorin Tirc

                    66210




                    66210






















                        up vote
                        2
                        down vote













                        From the power mean inequality we know that



                        $$sqrt[q]{sum_{i=1}^nw_ix_i^q} geq sqrt[p]{sum_{i=1}^nw_ix_i^p} quad forall p <q$$



                        where $ sum_{i=1}^n w_i=1 $



                        In your case $p=2, q=3$ and $n=3$. Hence,



                        $$left(dfrac{a^3+b^3+c^3}{3}right)^{1/3}ge left(dfrac{a^2+b^2+c^2}{3}right)^{1/2}=3$$






                        share|cite|improve this answer

























                          up vote
                          2
                          down vote













                          From the power mean inequality we know that



                          $$sqrt[q]{sum_{i=1}^nw_ix_i^q} geq sqrt[p]{sum_{i=1}^nw_ix_i^p} quad forall p <q$$



                          where $ sum_{i=1}^n w_i=1 $



                          In your case $p=2, q=3$ and $n=3$. Hence,



                          $$left(dfrac{a^3+b^3+c^3}{3}right)^{1/3}ge left(dfrac{a^2+b^2+c^2}{3}right)^{1/2}=3$$






                          share|cite|improve this answer























                            up vote
                            2
                            down vote










                            up vote
                            2
                            down vote









                            From the power mean inequality we know that



                            $$sqrt[q]{sum_{i=1}^nw_ix_i^q} geq sqrt[p]{sum_{i=1}^nw_ix_i^p} quad forall p <q$$



                            where $ sum_{i=1}^n w_i=1 $



                            In your case $p=2, q=3$ and $n=3$. Hence,



                            $$left(dfrac{a^3+b^3+c^3}{3}right)^{1/3}ge left(dfrac{a^2+b^2+c^2}{3}right)^{1/2}=3$$






                            share|cite|improve this answer












                            From the power mean inequality we know that



                            $$sqrt[q]{sum_{i=1}^nw_ix_i^q} geq sqrt[p]{sum_{i=1}^nw_ix_i^p} quad forall p <q$$



                            where $ sum_{i=1}^n w_i=1 $



                            In your case $p=2, q=3$ and $n=3$. Hence,



                            $$left(dfrac{a^3+b^3+c^3}{3}right)^{1/3}ge left(dfrac{a^2+b^2+c^2}{3}right)^{1/2}=3$$







                            share|cite|improve this answer












                            share|cite|improve this answer



                            share|cite|improve this answer










                            answered Nov 16 at 13:39









                            callculus

                            17.6k31427




                            17.6k31427






















                                up vote
                                2
                                down vote













                                Okay, the question requires knowledge of the AM-GM inequality, which you do have.
                                You need to apply it twice; first on $a^2,b^2,c^2$ and obtain an inequality on the product $abc$; then on $a^3,b^3,c^3$ and substitute $abc$.
                                The answer is $81$.
                                Hope this helps.






                                share|cite|improve this answer



























                                  up vote
                                  2
                                  down vote













                                  Okay, the question requires knowledge of the AM-GM inequality, which you do have.
                                  You need to apply it twice; first on $a^2,b^2,c^2$ and obtain an inequality on the product $abc$; then on $a^3,b^3,c^3$ and substitute $abc$.
                                  The answer is $81$.
                                  Hope this helps.






                                  share|cite|improve this answer

























                                    up vote
                                    2
                                    down vote










                                    up vote
                                    2
                                    down vote









                                    Okay, the question requires knowledge of the AM-GM inequality, which you do have.
                                    You need to apply it twice; first on $a^2,b^2,c^2$ and obtain an inequality on the product $abc$; then on $a^3,b^3,c^3$ and substitute $abc$.
                                    The answer is $81$.
                                    Hope this helps.






                                    share|cite|improve this answer














                                    Okay, the question requires knowledge of the AM-GM inequality, which you do have.
                                    You need to apply it twice; first on $a^2,b^2,c^2$ and obtain an inequality on the product $abc$; then on $a^3,b^3,c^3$ and substitute $abc$.
                                    The answer is $81$.
                                    Hope this helps.







                                    share|cite|improve this answer














                                    share|cite|improve this answer



                                    share|cite|improve this answer








                                    edited Nov 17 at 13:16

























                                    answered Nov 16 at 13:23









                                    AryanSonwatikar

                                    759




                                    759






















                                        up vote
                                        0
                                        down vote













                                        For $a=b=c=3$ we obtain a value $81$.



                                        We'll prove that it's a minimal value.



                                        Indeed, we need to prove that $$a^3+b^3+c^3geq81,$$ which is true because
                                        $$a^3+b^3+c^3-81=sum_{cyc}(a^3-27)=$$
                                        $$=sum_{cyc}left(a^3-27-frac{9}{2}(a^2-9)right)=frac{1}{2}sum_{cyc}(a-3)^2(2a+3)geq0.$$
                                        Also, Holder helps:
                                        $$a^3+b^3+c^3=sqrt{frac{(a^3+b^3+c^3)^2(1+1+1)}{3}}geqsqrt{frac{(a^2+b^2+c^2)^3}{3}}=81.$$






                                        share|cite|improve this answer























                                        • From which source can I understand holder's inequality with elementary illustrations and exercises. You seem to be good at this.
                                          – Ramanujam Ganit Prashikshan Ke
                                          Nov 16 at 16:55










                                        • @Ramanujam Ganit Prashikshan Ke You can read solutions with tag holder-inequality in this forum. I think it's the best illustration. See also here: math.stackexchange.com/tags/holder-inequality/info
                                          – Michael Rozenberg
                                          Nov 16 at 17:29

















                                        up vote
                                        0
                                        down vote













                                        For $a=b=c=3$ we obtain a value $81$.



                                        We'll prove that it's a minimal value.



                                        Indeed, we need to prove that $$a^3+b^3+c^3geq81,$$ which is true because
                                        $$a^3+b^3+c^3-81=sum_{cyc}(a^3-27)=$$
                                        $$=sum_{cyc}left(a^3-27-frac{9}{2}(a^2-9)right)=frac{1}{2}sum_{cyc}(a-3)^2(2a+3)geq0.$$
                                        Also, Holder helps:
                                        $$a^3+b^3+c^3=sqrt{frac{(a^3+b^3+c^3)^2(1+1+1)}{3}}geqsqrt{frac{(a^2+b^2+c^2)^3}{3}}=81.$$






                                        share|cite|improve this answer























                                        • From which source can I understand holder's inequality with elementary illustrations and exercises. You seem to be good at this.
                                          – Ramanujam Ganit Prashikshan Ke
                                          Nov 16 at 16:55










                                        • @Ramanujam Ganit Prashikshan Ke You can read solutions with tag holder-inequality in this forum. I think it's the best illustration. See also here: math.stackexchange.com/tags/holder-inequality/info
                                          – Michael Rozenberg
                                          Nov 16 at 17:29















                                        up vote
                                        0
                                        down vote










                                        up vote
                                        0
                                        down vote









                                        For $a=b=c=3$ we obtain a value $81$.



                                        We'll prove that it's a minimal value.



                                        Indeed, we need to prove that $$a^3+b^3+c^3geq81,$$ which is true because
                                        $$a^3+b^3+c^3-81=sum_{cyc}(a^3-27)=$$
                                        $$=sum_{cyc}left(a^3-27-frac{9}{2}(a^2-9)right)=frac{1}{2}sum_{cyc}(a-3)^2(2a+3)geq0.$$
                                        Also, Holder helps:
                                        $$a^3+b^3+c^3=sqrt{frac{(a^3+b^3+c^3)^2(1+1+1)}{3}}geqsqrt{frac{(a^2+b^2+c^2)^3}{3}}=81.$$






                                        share|cite|improve this answer














                                        For $a=b=c=3$ we obtain a value $81$.



                                        We'll prove that it's a minimal value.



                                        Indeed, we need to prove that $$a^3+b^3+c^3geq81,$$ which is true because
                                        $$a^3+b^3+c^3-81=sum_{cyc}(a^3-27)=$$
                                        $$=sum_{cyc}left(a^3-27-frac{9}{2}(a^2-9)right)=frac{1}{2}sum_{cyc}(a-3)^2(2a+3)geq0.$$
                                        Also, Holder helps:
                                        $$a^3+b^3+c^3=sqrt{frac{(a^3+b^3+c^3)^2(1+1+1)}{3}}geqsqrt{frac{(a^2+b^2+c^2)^3}{3}}=81.$$







                                        share|cite|improve this answer














                                        share|cite|improve this answer



                                        share|cite|improve this answer








                                        edited Nov 16 at 15:21

























                                        answered Nov 16 at 15:14









                                        Michael Rozenberg

                                        94.3k1588183




                                        94.3k1588183












                                        • From which source can I understand holder's inequality with elementary illustrations and exercises. You seem to be good at this.
                                          – Ramanujam Ganit Prashikshan Ke
                                          Nov 16 at 16:55










                                        • @Ramanujam Ganit Prashikshan Ke You can read solutions with tag holder-inequality in this forum. I think it's the best illustration. See also here: math.stackexchange.com/tags/holder-inequality/info
                                          – Michael Rozenberg
                                          Nov 16 at 17:29




















                                        • From which source can I understand holder's inequality with elementary illustrations and exercises. You seem to be good at this.
                                          – Ramanujam Ganit Prashikshan Ke
                                          Nov 16 at 16:55










                                        • @Ramanujam Ganit Prashikshan Ke You can read solutions with tag holder-inequality in this forum. I think it's the best illustration. See also here: math.stackexchange.com/tags/holder-inequality/info
                                          – Michael Rozenberg
                                          Nov 16 at 17:29


















                                        From which source can I understand holder's inequality with elementary illustrations and exercises. You seem to be good at this.
                                        – Ramanujam Ganit Prashikshan Ke
                                        Nov 16 at 16:55




                                        From which source can I understand holder's inequality with elementary illustrations and exercises. You seem to be good at this.
                                        – Ramanujam Ganit Prashikshan Ke
                                        Nov 16 at 16:55












                                        @Ramanujam Ganit Prashikshan Ke You can read solutions with tag holder-inequality in this forum. I think it's the best illustration. See also here: math.stackexchange.com/tags/holder-inequality/info
                                        – Michael Rozenberg
                                        Nov 16 at 17:29






                                        @Ramanujam Ganit Prashikshan Ke You can read solutions with tag holder-inequality in this forum. I think it's the best illustration. See also here: math.stackexchange.com/tags/holder-inequality/info
                                        – Michael Rozenberg
                                        Nov 16 at 17:29




















                                         

                                        draft saved


                                        draft discarded



















































                                         


                                        draft saved


                                        draft discarded














                                        StackExchange.ready(
                                        function () {
                                        StackExchange.openid.initPostLogin('.new-post-login', 'https%3a%2f%2fmath.stackexchange.com%2fquestions%2f3001046%2fis-it-based-on-tchebyshev%23new-answer', 'question_page');
                                        }
                                        );

                                        Post as a guest















                                        Required, but never shown





















































                                        Required, but never shown














                                        Required, but never shown












                                        Required, but never shown







                                        Required, but never shown

































                                        Required, but never shown














                                        Required, but never shown












                                        Required, but never shown







                                        Required, but never shown







                                        Popular posts from this blog

                                        How do I know what Microsoft account the skydrive app is syncing to?

                                        Grease: Live!

                                        When does type information flow backwards in C++?